Nice question to practice power & roots

This topic has expert replies
Master | Next Rank: 500 Posts
Posts: 118
Joined: Wed Mar 16, 2011 12:51 am
Location: New Delhi, India
Thanked: 12 times
Followed by:1 members

Nice question to practice power & roots

by pankajks2010 » Sat Apr 09, 2011 4:38 am
Hi,

read this piece by Vivian Kerr on power and roots:
https://www.beatthegmat.com/mba/2011/04/ ... ter-scores

Here's a DS question to practice it further:

Is 1 > |x-1|?

a)(x-1)^2 <= 1
b) x^2-1 > 0

User avatar
Legendary Member
Posts: 1101
Joined: Fri Jan 28, 2011 7:26 am
Thanked: 47 times
Followed by:13 members
GMAT Score:640

by HSPA » Sat Apr 09, 2011 5:21 am
a) looks okay
b) x> -1 and x>1 so x>1 is the result

x = 2,3,4,5....
1 > |2-1| No.. 1>|3-1|No...

D for me
First take: 640 (50M, 27V) - RC needs 300% improvement
Second take: coming soon..
Regards,
HSPA.

Master | Next Rank: 500 Posts
Posts: 118
Joined: Wed Mar 16, 2011 12:51 am
Location: New Delhi, India
Thanked: 12 times
Followed by:1 members

by pankajks2010 » Sat Apr 09, 2011 5:29 am
HSPA wrote:a) looks okay
b) x> -1 and x>1 so x>1 is the result

x = 2,3,4,5....
1 > |2-1| No.. 1>|3-1|No...

D for me
HSPA...2 mistakes in 2 questions..come on boy..The correct answer is E.

Now just try the question again and if required go through the note written by Vivien..

User avatar
Master | Next Rank: 500 Posts
Posts: 436
Joined: Tue Feb 08, 2011 3:07 am
Thanked: 72 times
Followed by:6 members

by manpsingh87 » Sat Apr 09, 2011 5:30 am
pankajks2010 wrote:Hi,

read this piece by Vivian Kerr on power and roots:
https://www.beatthegmat.com/mba/2011/04/ ... ter-scores

Here's a DS question to practice it further:

Is 1 > |x-1|?

a)(x-1)^2 <= 1
b) x^2-1 > 0
lets just analyze |x-1|<1;
when x>1 |x-1|=(x-1), therefore (x-1)<1; x<2 therefore |x-1| will be less than 1 when x lies between 1 and 2 i.e. 1<x<2,

now when x<1, |x-1|=-(x-1), -x+1<1, x>0; hence |x-1| will be less than 1 when 0<x<1;

so, |x-1|<1 will be true for 1<x<2 and 0<x<1;

now consider statements 1 and 2:
1)(x-1)^2<=1; x(x-1)<=0 x<=0, x<=2, for x<=0 |x-1| doesn't holds true and for x<=2, consider two cases when x=2, |x-1|<1 won't hold true and for x<2, for some values it holds true (which lies in the two ranges 1<x<2 and 0<x<1) and for some it won't hence 1 alone is not sufficient to answer the question.

2)x^2-1>0, (x-1)(x+1)>0; x>1 and x>-1, similary here also different results are possible hence 2 alone is also not sufficient to answer the question.

combining 1 and 2 we have 1<x<=2, now the inequality |x-1|<1 holds true for all values except x=2,
and for the other values of x |x-1|<1 won't hold true hence answer should be E
O Excellence... my search for you is on... you can be far.. but not beyond my reach!

User avatar
Legendary Member
Posts: 1101
Joined: Fri Jan 28, 2011 7:26 am
Thanked: 47 times
Followed by:13 members
GMAT Score:640

by HSPA » Sat Apr 09, 2011 5:40 am
Oh my god.. enough math threads for today.. sorry as I think that I am brain dead for today, reading from morning on diff topics...

I saw... |x| = sqrt(x^2) but there is only x^2 ... :(
First take: 640 (50M, 27V) - RC needs 300% improvement
Second take: coming soon..
Regards,
HSPA.

Master | Next Rank: 500 Posts
Posts: 118
Joined: Wed Mar 16, 2011 12:51 am
Location: New Delhi, India
Thanked: 12 times
Followed by:1 members

by pankajks2010 » Sat Apr 09, 2011 6:03 am
first statement: (x-1)^2<=1 ---> -1<=(x-1)<=1 ---> 0<=x<=2
Now except for x=2, every value between 0-1.9 satisfies the required condition. Thus, first statement is insufficient

second statement: x^2-1>0 ----> either x<-1 or x>1
now, again by substituting different values (in the range, x<-1 or x>1) we get different results. Thus, this too is insufficient.

Thus, E

User avatar
GMAT Instructor
Posts: 15539
Joined: Tue May 25, 2010 12:04 pm
Location: New York, NY
Thanked: 13060 times
Followed by:1906 members
GMAT Score:790

by GMATGuruNY » Sat Apr 09, 2011 6:31 am
pankajks2010 wrote:Hi,

read this piece by Vivian Kerr on power and roots:
https://www.beatthegmat.com/mba/2011/04/ ... ter-scores

Here's a DS question to practice it further:

Is 1 > |x-1|?

a)(x-1)^2 <= 1
b) x^2-1 > 0
Rephrase the question stem:

|x-1| < 1
-1 < x-1 < 1
0 < x < 2

Question rephrased: Is 0 < x < 2?

Try to plug in values that satisfy both statements.

Statement 1: (x-1)² ≤ 1.
x = 2 works, because (2-1)² ≤ 1.
Is 0 < 2 < 2? No.

x = 1.5 works, because (1.5-1)² ≤ 1.
Is 0 < 1.5 < 2? Yes.

Since the answer can be both No and Yes, insufficient.

Statement 2: x²-1 > 0.
The values that were used in statement 1 -- x=2 and x=1.5 -- also satisfy statement 2.
Since these values showed that statement 1 was insufficient. they also show that statement 2 is insufficient.

Statements 1 and 2 combined:
Since x=2 and x=1.5 satisfy both statements, we know that even when we combine the 2 statements, the answer to the question stem can be both No and Yes.

The correct answer is E.
Private tutor exclusively for the GMAT and GRE, with over 20 years of experience.
Followed here and elsewhere by over 1900 test-takers.
I have worked with students based in the US, Australia, Taiwan, China, Tajikistan, Kuwait, Saudi Arabia -- a long list of countries.
My students have been admitted to HBS, CBS, Tuck, Yale, Stern, Fuqua -- a long list of top programs.

As a tutor, I don't simply teach you how I would approach problems.
I unlock the best way for YOU to solve problems.

For more information, please email me (Mitch Hunt) at [email protected].
Student Review #1
Student Review #2
Student Review #3